How to use the normal form of the Green's Theorem?

Click For Summary
SUMMARY

The discussion focuses on applying Green's Theorem to compute the flux integral of the vector field F, defined as the gradient of the scalar potential function f(x, y) = 1/2(x² + y²). The correct approach involves calculating the gradient F = ∇f, which yields F(x, y) = (x, y). The flux integral over the positively oriented unit circle C, parametrized by r(t) = (cos t, sin t), results in a value of 2π, confirming the application of the theorem. The initial incorrect attempt involved misidentifying the components of F, leading to an erroneous result of 0.

PREREQUISITES
  • Understanding of Green's Theorem and its application in vector calculus.
  • Knowledge of scalar potential functions and their gradients.
  • Familiarity with parametrization of curves, specifically the unit circle.
  • Ability to compute partial derivatives of functions of multiple variables.
NEXT STEPS
  • Learn how to compute the gradient of scalar functions in multivariable calculus.
  • Study the applications of Green's Theorem in various contexts, including flux integrals.
  • Explore examples of parametrizing different curves in the plane.
  • Investigate the relationship between line integrals and double integrals in vector fields.
USEFUL FOR

Students studying vector calculus, particularly those learning about Green's Theorem, as well as educators and tutors seeking to clarify concepts related to flux integrals and scalar potential functions.

randomcat
Messages
7
Reaction score
0

Homework Statement


Suppose that F = ∇f for some scalar potential function f(x, y) = 1/2(x2 + y2)
Let C denote the positively oriented unit circle, parametrized by r(t) = (cos t, sin t), 0 ≤ t ≤ 2∏. Compute the flux integral of \ointF\bulletN ds, where N is the outward unit normal to C.

Homework Equations


I know that the theorem is basically \ointF\bulletN ds = double integral of (\frac{\partial M}{\partial x} + \frac{\partial N}{\partial y}) dx dy

My question is, how do you find F(x, y)? I'm given f(x, y) in the problem, but do I need to use it to solve for the answer?

The Attempt at a Solution


What I did (which was wrong) was that I set M = 1/2x2 and N = 1/2y2 and took the partial derivatives of those. That gives me (x, y). With the parametrization, I get (cos t, sin t). Then I integrate them over the unit circle and got 0, which was wrong. The correct answer is 2pi.

My professor wrote that F = ∇f = F(x, y) in the solutions, but where did F(x, y) come from?
 
Physics news on Phys.org
randomcat said:

Homework Statement


Suppose that F = ∇f for some scalar potential function f(x, y) = 1/2(x2 + y2)
Let C denote the positively oriented unit circle, parametrized by r(t) = (cos t, sin t), 0 ≤ t ≤ 2∏. Compute the flux integral of \ointF\bulletN ds, where N is the outward unit normal to C.


Homework Equations


I know that the theorem is basically \ointF\bulletN ds = double integral of (\frac{\partial M}{\partial x} + \frac{\partial N}{\partial y}) dx dy

My question is, how do you find F(x, y)? I'm given f(x, y) in the problem, but do I need to use it to solve for the answer?

The Attempt at a Solution


What I did (which was wrong) was that I set M = 1/2x2 and N = 1/2y2 and took the partial derivatives of those. That gives me (x, y). With the parametrization, I get (cos t, sin t). Then I integrate them over the unit circle and got 0, which was wrong. The correct answer is 2pi.

My professor wrote that F = ∇f = F(x, y) in the solutions, but where did F(x, y) come from?

You are given ##f=(x^2+y^2)/2## and ##F=\nabla f##. So calculate ##\nabla f## to get ##F##.
 
Perhaps the problem is that you don't know what \nabla f means? If f is as function of two variables, x and y, the \nabla f is the vector function
\frac{\partial f}{\partial x}\vec{i}+ \frac{\partial f}{\partial y}\vec{j}

That should be easy to calculate for this f.
 
Question: A clock's minute hand has length 4 and its hour hand has length 3. What is the distance between the tips at the moment when it is increasing most rapidly?(Putnam Exam Question) Answer: Making assumption that both the hands moves at constant angular velocities, the answer is ## \sqrt{7} .## But don't you think this assumption is somewhat doubtful and wrong?

Similar threads

Replies
3
Views
2K
  • · Replies 8 ·
Replies
8
Views
2K
Replies
12
Views
2K
  • · Replies 7 ·
Replies
7
Views
2K
  • · Replies 10 ·
Replies
10
Views
2K
  • · Replies 6 ·
Replies
6
Views
2K
  • · Replies 1 ·
Replies
1
Views
2K
Replies
4
Views
1K
Replies
5
Views
2K
  • · Replies 11 ·
Replies
11
Views
3K